7
$\begingroup$

Is it possible to have a non-split short exact sequences of vector bundles (on some smooth variety) $0\rightarrow V_1 \rightarrow V_2 \rightarrow V_3 \rightarrow 0$. Such that $V_2\cong V_1\oplus V_3$ as vector bundles?

$\endgroup$
6
  • 1
    $\begingroup$ What do you mean by 'as objects'? $\endgroup$
    – Jef
    Nov 17, 2020 at 18:49
  • 2
    $\begingroup$ As vector bundles. Not as extensions (which obviously is impossible according to the assumptions). $\endgroup$
    – user127776
    Nov 17, 2020 at 18:50
  • 1
    $\begingroup$ See also mathoverflow.net/questions/163041/… . $\endgroup$
    – LSpice
    Nov 17, 2020 at 19:24
  • 4
    $\begingroup$ You can take a non-split short exact sequence on $\mathbb{P}^1$ of the form $$ 0 \to \mathcal{O} \to \mathcal{O} \oplus \mathcal{O}(n) \to \mathcal{O}(n) \to 0.$$ It exists as soon as $n \geq 2$, as one can check by computing the Ext$^1$ group. $\endgroup$ Nov 17, 2020 at 19:26
  • 5
    $\begingroup$ @FrancescoPolizzi : but for example, there is a non-split sequence $0 \to \mathcal O \to \mathcal O(1)^{\oplus 2} \to \mathcal O(2) \to 0$ corresponding to a non-zero element in $Ext^1(\mathcal O(2), \mathcal O)$. So given a non-zero class in $Ext^1(\mathcal O, \mathcal O(n))$ do you know that the middle term will be given by $\mathcal O \oplus \mathcal O(n)$ ? $\endgroup$ Nov 17, 2020 at 20:06

1 Answer 1

11
$\begingroup$

$\newcommand{\cO}{\mathcal{O}}$Consider exact sequence of trivial vector bundles $$0\to\cO\xrightarrow{\left(\begin{matrix}x \\ y\end{matrix}\right)}\cO\oplus\cO\xrightarrow{\left(\begin{matrix}y & -x\end{matrix}\right)}\cO\to 0$$ on $X=\mathbb{A}^2_{x,y}\setminus\{0\}$. One checks easily that it is exact on stalks (but the same sequence on $\mathbb{A}^2$ is not exact at $(x,y)=(0,0)$). It is the pullback of $0\to\cO(-1)\to \cO\oplus\cO\to \cO(1)\to 0$ from $\mathbb{P}^1$. A splitting of this sequence would be a pair of functions $f_1,f_2\in H^0(\mathbb{A}^2\setminus\{0\},\cO)=k[x,y]$ such that $yf_1-xf_2=1$ but there is no such pair.


On the positive side, any such sequence has to be split if $X$ is proper. In this case the spaces $Hom(V_3, V_1)$ and $Hom(V_3, V_3)$ are finite-dimensional over $k$. Applying $Hom(V_3,-)$ to this exact sequence we get a left exact sequence of finite-dimensional vector spaces $$0\to Hom(V_3, V_1)\to Hom(V_3,V_2)\to Hom(V_3,V_3)$$ The dimension of the vector space in the middle $Hom(V_3,V_2)\simeq Hom(V_3, V_1\oplus V_3)$ is equal to the sum of the dimensions of first and third terms. Therefore, the sequence has to be exact on the right and, in particular, the identity $Id_{V_3}\in Hom(V_3, V_3)$ lifts to a morphism $Hom(V_3, V_2)$ that gives a splitting.

$\endgroup$
5
  • $\begingroup$ Yes my $X$ is proper. Your argument is super-smart. So there are no examples of such thing on projective line. This refutes one of the comments above. Thanks. $\endgroup$
    – user127776
    Nov 17, 2020 at 20:11
  • 1
    $\begingroup$ @user127776 : I think the explanation is given by my comment below $\endgroup$ Nov 17, 2020 at 20:11
  • $\begingroup$ Does this really refute my example? My middle term is not balanced, how do you reduce yourself to the case of the sum of two trivial bundles? $\endgroup$ Nov 17, 2020 at 20:37
  • 2
    $\begingroup$ The explanation given above for proper variety is general. It is not specialized to the specific short exact sequence of trivial bundles given at the start. $\endgroup$
    – user127776
    Nov 17, 2020 at 21:00
  • $\begingroup$ Ok, I see. I was confused by "any such sequence" at the beginning of the second part of the answer. $\endgroup$ Nov 17, 2020 at 21:50

Your Answer

By clicking “Post Your Answer”, you agree to our terms of service and acknowledge that you have read and understand our privacy policy and code of conduct.

Not the answer you're looking for? Browse other questions tagged or ask your own question.